Các phương pháp chứng minh bất đẳng thức: Phần 2 - Nguyễn Tất Thu

90 8 0
Các phương pháp chứng minh bất đẳng thức: Phần 2 - Nguyễn Tất Thu

Đang tải... (xem toàn văn)

Tài liệu hạn chế xem trước, để xem đầy đủ mời bạn chọn Tải xuống

Thông tin tài liệu

Nối tiếp nội dung phần 1, phần 2 tài liệu Các phương pháp chứng minh bất đẳng thức sẽ cung cấp tới người học về đáp số và hướng dẫn giải các bài toán về chứng minh bất đẳng thức. Thông qua việc tham khảo và luyện tập giải bài, các em học sinh sẽ nắm vững nội dung bài học và có phương pháp học tập hiệu quả. Mời các bạn cùng tham khảo.

2 BÀI TỐN TÌM HẰNG SỐ TỐT NHẤT TRONG BẤT ĐẲNG THỨC ĐÁP SỐ VÀ HƯỚNG DẪN GIẢI 85 Chương Các bất đẳng thức cổ điển §1 Bất đẳng thức AM-GM Câu 1.1 a) Bất đẳng thức cần chứng minh tương đương với 1.1.1 + (1 + a) (1 + b) (1 + c) abc ≤ (1 + a) (1 + b) (1 + c) Đặt : T = T ≤ 1.1.1 + (1 + a) (1 + b) (1 + c) abc (1 + a) (1 + b) (1 + c) 1 1 a b c + + + + + 1+a 1+b 1+c 1+a 1+b 1+c T ≤ a+1 b+1 c+1 + + = = 1+a 1+b 1+c Dấu đẳng thức xảy a = b = c ≥ b) Ta có a b c a a b c a c b =2+ + + + + + b c a c b a a+b b+c c+a +1 + +1 + +1 −1 = c a b 1 = (a + b + c) + + −1 a b c 3(a + b + c) 2(a + b + c) √ √ ≥ −1≥ + 3 abc abc 1+ 1+ b c 1+ Câu 1.2 Bất đẳng thức cần chứng minh tương đương với n (1 + a1 )(1 + a2 ) · · · (1 + an ) + n a1 a2 · · · an ≤ (1 + a1 )(1 + a2 ) · · · (1 + an ) Áp dụng bất đẳng thức AM-GM ta có V T (1) ≤ n i=1 1 + + n 86 i=1 = 1 + (1) BẤT ĐẲNG THỨC AM-GM Bài toán chứng minh Câu 1.3 Ta có abc ≤ a+b+c = 27 Khi 1+ a 1+ b 1+ c ≥ 1+ √ abc ≥ 64 Suy (1 + a) (1 + b) (1 + c) ≥ 64abc Câu 1.4 Bất đẳng thức cần chứng minh tương đương với n a1 a2 · · · an + (a1 + b1 )(a2 + b2 ) · · · (an + bn ) n b1 b2 · · · bn ≤ (a1 + b1 )(a2 + b2 ) · · · (an + bn ) (1) Áp dụng bất đẳng thức AM-GM ta có n n a1 a2 · · · an ≤ (a1 + b1 )(a2 + b2 ) · · · (an + bn ) n a1 an + ··· + a1 + b1 an + bn b1 b2 · · · bn ≤ (a1 + b1 )(a2 + b2 ) · · · (an + bn ) n b1 bn + ··· + a1 + b1 an + bn Cộng hai bất đẳng thức theo vế ta có đpcm n αi = nên tồn số nguyên dương N,k1 ,k2 , · · · , kn Câu 1.5 Vì αi số hữu tỉ dương i=1 ki Áp dụng bất đẳng thức AM-GM cho N số, ta có N a1 + a1 + · · · + a1 + · · · + an + an + · · · + an k1 kn k1 số kn số ≥ a1n · · · ann = aα1 · · · aαnn αi · = N cho αi = n i=1 Bất đẳng thức chứng minh Câu 1.6 Chuẩn hóa a1 + a2 + · · · + an = n, ta cần chứng minh ak1 + ak2 + · · · akn ≥ n (1) Áp dụng bất đẳng thức AM − GM cho k số gồm k − số aki ta có n aki + k − ≥ kai ⇒ n aki + n(k − 1) ≥ k i=1 n aki ≥ n = kn ⇒ i=1 i=1 Vậy (1) đúng, hay toán chứng minh Câu 1.7 Áp dụng bất đẳng thức 1 + ≥ ta có x y x+y 1 + ≥ = a + 3b b + 2c + a (a + 3b) + (b + 2c + a) a + 2b + c 1 + ≥ b + 3c 2a + b + c a + b + 2c 1 + ≥ c + 3a a + 2b + c 2a + b + c Cộng bất đẳng thức theo vế ta có đpcm 87 BẤT ĐẲNG THỨC AM-GM Câu 1.8 Áp dụng bất đẳng thức Cô si cho hai số ta có √ a+ √ √ √ √ b ≥ ab ⇒ a + b Suy √ a+ √ Dẫn tới √ √ a+ 4b √ 1 ≥ 16 ab + ≥ √ a b ab √ ≥ 16 ab √ = 32 ab 1 + a b b ≤ 32 1 + a b Tương tự: √ b+ √ ≤ c 32 1 + b c 1 , √ ≤ √ 32 ( c + a) 1 + c a Cộng bất đẳng thức ta có √ √ a+ 4b + √ √ b+ 4c 1 ≤ + √ √ 16 ( c + a) 1 + + a b c Mặt khác, theo giả thiết ta có ab + bc + ca ≤ 3abc nên suy 1 + + ≤ a b c Suy √ √ a+ 4b + √ b+ √ 4 c + √ (đpcm) √ ≤ 4 16 ( c + a) Câu 1.9 Bất đẳng thức cần chứng minh tương đương với 2b 2c 2a 2b 2c 2a + + ≥4− − − b + 2c c + 2a a + 2b b + 2a c + 2b a + 2c 1 1 1 ⇔a + +b + +c + b + 2c b + 2a c + 2a c + 2b a + 2b a + 2c Áp dụng bất đẳng thức ≥ 1 + ≥ ta có x y x+y 1 + ≥ = b + 2c b + 2a 2a + 2b + 2c a+b+c Suy a 1 + b + 2c b + 2a ≥ 2a a+b+c Tương tự: b 1 + c + 2a c + 2b ≥ 2b ,c a+b+c 1 + a + 2b a + 2c ≥ 2c a+b+c Cộng bất đẳng thức ta có a 1 + b + 2c b + 2a +b 1 + c + 2a c + 2b 88 +c 1 + a + 2b a + 2c ≥ (đpcm) BẤT ĐẲNG THỨC AM-GM Câu 1.10 Áp dụng AM – GM, ta có + x3 = (1 + x) − x + x2 ≤ (1 + x + − x + x2 ) (2 + x2 ) = 4 Tương tự 1 + y3 ≥ 2 √ ; ≥ + y2 + z2 + z3 Vậy P =√ + + x3 1 + y3 +√ 2 ≥ + + 2 2+x 2+y + z2 + z3 Áp dụng Cauchy – Swarzt, ta được: P ≥ 18 ≥ x2 + y + z + Dấu ‘=’ xảy x = y = z = Vậy GTNN biểu thức P = Câu 1.11 Ta có: a a (1 + b2 ) − ab2 ab2 ab2 ab = = a − ≥ a − = a − + b2 + b2 + b2 2b Do đó: b c a + + ≥ a + b + c − (ab + bc + ca) 2 1+b 1+c 1+a Mà: ab + bc + ca ≤ (a + b + c)2 = 3 Nên suy ra: a b c 3 + + ≥ − = + b2 + c2 + a2 2 Câu 1.12 Ta có: a2 a (a + 2b2 ) − 2ab2 2ab2 2ab2 √ √ √ = = a − ≥ a − = a − b a a + 2b2 a + 2b2 a + b2 + b2 2ab Suy ra: √ √ a2 b2 c2 √ √ + + ≥ a + b + c − a.b + b.c + c.a a + 2b2 b + 2c2 c + 2a2 Mặt khác: ab + bc + ca ≤ Và √ √ √ √ √ √ ab b + bc c + ca a ≤ (a + b + c)2 = (ab + bc + ca) (a + b + c) ≤ √ 2 Vậy: a2 b2 c2 3 + + ≥ − = 2 a + 2b b + 2c c + 2a 4 89 BẤT ĐẲNG THỨC AM-GM Câu 1.13 ax + ay ≥ 2axy Đẳng thức xảy x = y √ by + cz ≥ √bcyz.Đẳng thức xảy by = cz cz + bx2 ≥ cbzx Đẳng thức xảy cz = bx2 Bây ta chọn a, b, c cho :    a + b =  a =   2c = ⇔ b=2   √   c = a = bc Suy ra: x2 + y ≥ 2xy Đẳng thức xảy x = y 1 2y + z ≥ 2yz.Đẳng thức xảy 2y = z 2 2 2 z + 2x ≥ 2zx Đẳng thức xảy z = 2x2 2 Cộng vế theo vế ta : 3x2 + 3y + z ≥ (xy + yz + zx) ⇒ 3x2 + 3y + z ≥ 10 (đpcm)  x=y       2y = z x=y=1 Đẳng thức xảy : ⇔  z=2  z = 2x      xy + yz + zx = Câu 1.14 Áp dụng bất đẳng thức Cô si cho số thực dương ta có a3 a + 2b b + 2c a3 a + 2b b + 2c + + ≥33 = a (a + 2b) (b + 2c) 27 27 (a + 2b) (b + 2c) 27 27 Tương tự: b3 b + 2c c + 2a + + ≥ b, (b + 2c) (c + 2a) 27 27 c3 c + 2a a + 2b + + ≥ c (c + 2a) (a + 2b) 27 27 Cộng ba bất đẳng thức ta có b3 c3 2(a + b + c) a+b+c a3 + + + ≥ (a + 2b) (b + 2c) (b + 2c) (c + 2a) (c + 2a) (a + 2b) Suy a3 b3 c3 a+b+c + + ≥ (a + 2b) (b + 2c) (b + 2c) (c + 2a) (c + 2a) (a + 2b) Đẳng thức xảy a = b = c Câu 1.15 Ta thấy đẳng thức xảy a = b = c = a4 Khi = ,b = 1,c + = nên áp dụng bất đẳng thức Cô si cho số ta b (c + 2) a4 b b c+2 a4 b b c+2 4 + + + ≥ + = a b2 (c + 2) 3 b2 (c + 2) 3 90 BẤT ĐẲNG THỨC AM-GM Tương tự: b4 2b a + c4 2a b + + + ≥ b, + + ≥ c c2 (a + 2) a2 (b + 2) Cộng bất đẳng thức ta có a4 b4 c4 7(a + b + c) + (a + b + c) + + + ≥ b2 (c + 2) c2 (a + 2) a2 (b + 2) Hay a4 b4 c4 5(a + b + c) − + + ≥ b (c + 2) c (a + 2) a (b + 2) √ Mà a + b + c ≥ 3 abc = nên ta có a4 b4 c4 + + ≥ (đpcm) b2 (c + 2) c2 (a + 2) a2 (b + 2) Câu 1.16 Áp dụng bất đẳng thức a+b + c b+c + a √ c+a ≥√ b √ a =√ x+ √ y≤ (x + y), ta có : √ √ a b √ +√ c c 1 √ +√ c b √ √ √ √ b c c b √ +√ √ +√ +√ +√ a a a a 2 √ √ b 1 c 1 √ +√ +√ √ +√ +√ a c a 2 b 1 + ≥ , ta có : x y x+y √ √ b c 1 √ +√ +√ √ +√ +√ a c a 2 b Áp dụng bất đẳng thức √ a √ 1 √ +√ c b √ √ √ 2c 2a 2b √ +√ √ ≥√ √ +√ a+ c b+ c a+ b √ √ Áp dụng bất đẳng thức x + y ≤ (x + y), ta có : √ √ √ √ 2b 2a 2c 2a √ √ +√ √ ≥ + √ +√ a+ c b+ c a+ b (b + c) (a + c) c + a+b b + a+c =2 √ 2b + √ 2c (a + b) a b+c Câu 1.17 Áp dụng bất đẳng thức AM-GM ta có a4 b+2 + ≥2 b+2 a4 b + 2 = a2 b+2 Tương tự: b4 c+2 c4 a+2 + ≥ b2 , + ≥ c2 c+2 a+2 Cộng ba bất đẳng thức ta có a4 b4 c4 a+b+c+6 2 + + + ≥ a + b2 + c2 = b+2 c+2 a+2 Suy a4 b4 c4 12 − (a + b + c) + + ≥ b+2 c+2 a+2 91 BẤT ĐẲNG THỨC AM-GM Mặt khác: (a2 + b2 + c2 ) = a+b+c≤ nên suy b4 c4 12 − a4 + + ≥ = (đpcm) b+2 c+2 a+2 Đẳng thức xảy a = b = c = Câu 1.18 Bất đẳng thức cần chứng minh tương đương với +1 − a2 +1 − b2 +1 − c2 ≥ (a + b + c)2 Từ đề bài, ta suy a2 ,b2 ,c2 < Áp dụng bất đẳng thức Cơ si ta có: + − a2 ≥ − a2 (3 − a2 ) = 4, − a2 suy + ≥ a2 + 2 3−a Tương tự: 4 + ≥ b + 2, + ≥ c2 + 2 3−b 3−c Do +1 − a2 +1 − b2 +1 − c2 ≥ a2 + b2 + c2 + Mặt khác: (a + b + c)2 = a+b √ √ + 1.c 2 ≤3 (a + b)2 +1 c2 + Ta chứng minh (a + b)2 + ≤ a2 + 2 b2 + Khai triển rút gọn, bất đẳng thức (*) trở thành 2a2 b2 + a2 + b2 + ≥ 6ab Bất đẳng thức hiển nhiên vì: a2 b2 + ≥ 4ab, a2 + b2 ≥ 2ab Vậy toán chứng minh Câu 1.19 Ta có: b + c ≤ 2(b2 + c2 ) Suy a2 a2 + b2 + c2 √ − √ b2 + c2 ≥ b+c 2(b2 + c2 ) Tương tự: b2 a2 + b2 + c2 √ c2 a2 + b2 + c2 √ ≥ − √ c2 + a2 ; ≥ − √ a2 + b2 c+a a+b 2 2(c2 + a2 ) 2(a2 + b2 ) 92 (∗) BẤT ĐẲNG THỨC AM-GM Suy VT ≥ a2 + b + c2 √ Ta có: √ √ √ 1 1 √ a + b2 + b2 + c2 + c2 + a2 +√ +√ −√ a2 + b2 b2 + c2 c2 + a2 1 √ √ √ +√ +√ ≥√ 2 2 2 2 a +b b +c c +a a + b + b + c2 + c2 + a2 a2 + b2 + c2 ≥ Suy √ √ √ a + b2 + b2 + c2 + c2 + a2 √ √ √ a2 + b2 + b2 + c2 + c2 + a2 VT ≥ √ 2 Đặt t= √ a2 + b2 + √ b2 + c2 + √ √ √ √ √ √ √ c2 + a2 ≥ ab + bc + ca ≥ abc √ t ≥ abc nên từ giả thiết ta suy 54 √ √ √ − abc t = a2 + b2 + b2 + c2 + c2 + a2 = √ ≥√ − t 2 108 √ ⇔ t3 + 108t − 378 ≥ √ √ √ ⇔ t − t2 + 2t + 126 ≥ ⇔ t ≥ Suy √ 3 Suy V T ≥ √ = 2  a = b = c √ Đẳng thức xảy − a3 ⇔ 3 2a = √ a=b=c ⇔ a = b = c = a3 + 6a − = Câu 1.20 Nhận thấy đẳng thức xảy a = b = c = = + nên ta có đánh giá a2 + 2b2 + = a2 + b2 + b2 + + ≥ 2ab + 2b + Do đó: a2 1 ≤ + 2b + ab + b + Suy ra: a2 1 1 + + ≤ 2 + 2b + b + 2c + c + 2a + 1 + + ab + b + bc + c + ca + a + Vậy ta cần chứng minh: 1 + + ≤ ab + b + bc + c + ca + a + Bất đẳng thức hiển nhiên đẳng thức Để chứng minh ta thay c = vào vế ab trái biến đổi ta có đpcm Câu 1.21 Ta có √ a3 + b ≤ 1 =√ √ 2 a3 b 93 a · 1 ≤ √ b + a b =√ BẤT ĐẲNG THỨC AM-GM Câu 1.22 Ta có 2ab + a2 + b2 + c2 + ab + bc + ca (a + b)2 + (c + a) (c + b) ab + ≥ = 2 Suy VT ≥ 3+ (a + c)(b + c) (a + b)(a + c) (b + a)(b + c) + + (a + b)2 (b + c)2 (c + a)2 ≥ Câu 1.23 Áp dụng bất đẳng thức AM-GM ta có a+b+c=a+ b+c b+c (b + c)2 + ≥3 a , 2 suy 2a b+c ≥ 3a a+b+c Chứng minh tương tự, ta có 2b c+a ≥ 3b a+b+c 2c a+b ≥ 3c a+b+c Cộng ba bất đẳng thức theo vế ta có đpcm Câu 1.24 Bài tốn chứng minh cách sử dụng đánh giá sau: a2 + b2 a3 + b3 ≤ a+b a2 + b2 2ab =a+b− a+b a+b Như ta phải chứng minh: Chú ý rằng: bc ca ab + + + a + b + c ≥ a+b b+c c+a Áp dụng bất đẳng thức AM-GM với abc = 1,ta có ngay: 2ab a+b 2bc b+c 2ca c+a + + + + + ≥ a+b b+c c+a Vậy ta có điều phải chứng minh.Đẳng thức xảy a = b = c = Câu 1.25 Ta có 13a2 b2 c2 − 2abc − 27a2 b2 c2 − (abc + 2)2 27a2 b2 c2 = ≤ (a2 + b2 + c2 )3 2(a2 + b2 + c2 )3 2(a2 + b2 + c2 )3 Ta chứng minh: 27a2 b2 c2 a2 b2 c2 ≤ ⇔ 3 ≤ 2 2 2 54 2(a + b + c ) (a + b + c ) Vì a + b + c = nên ba số a,b,c có hai số dấu, ta giả sử hai số a,b Khi ab ≥ nên (a + b)2 (a + b)2 2 2 2 a +b +c = a + b + (a + b) = a + b + + 2 ≥ 27 27 a + b2 (a + b)2 (a + b)2 ≥ 2ab.4ab.c2 = 54a2 b2 c2 4 Suy a2 b2 c2 (đpcm) ≤ 2 54 (a + b + c ) 94 BÀI TỐN TÌM HẰNG SỐ TỐT NHẤT Do k − ≤ ⇔ k ≤ Ta chứng minh k = giá trị lớn cần tìm Tức ta cần chứng minh: b c a + + ≥ + 9bc + 4(b − c) + 9ca + 4(c − a) + 9ab + 4(a − b) (1) Áp dụng Bất đẳng thức Cauchy – Schwarz, ta có: (a + b + c)2 (a + b + c) + 27abc + 4a(b − c)2 + 4b(c − a)2 + 4c(a − b)2 = + 3abc + 4ab (a + b) + 4bc (b + c) + 4ca (c + a) VT ≥ Do để chứng minh (1) ta cần chứng minh: ≥ 3abc + 4ab (a + b) + 4bc (b + c) + 4ca (c + a) ⇔ (a + b + c)3 ≥ 3abc + 4ab (a + b) + 4bc (b + c) + 4ca (c + a) ⇔ a3 + b3 + c3 + 3abc ≥ ab (a + b) + bc (b + c) + ca (c + a) (điều đúng, bất đẳng thức Schur) Do (1) với a, b, c không âm thỏa a + b + c = với kmax = Vậy k = số cần tìm a Câu 2.3 Thử chọn b = c xét hàm theo biến , ta không dẫn đến kết cần tìm Như b ta thử chọn biến 0, biến tiến dần đến Trong bất đẳng thức, ta cho c → 0, a3 + b3 ≥ M ab2 Nhưng lại theo bất đẳng thức AM – GM, ta có b3 b3 a3 + b3 = a3 + + ≥ √ · ab2 2 3 Như ta thấy M ≤ √ Do ta chứng minh M = √ giá trị lớn cần tìm sau: 3 4 Khơng giảm tính tổng qt, giả sử c = {a,b,c} Đặt a = u + c, b = v + c với u, v ≥ Ta chứng minh: (u + c)3 + (v + c)3 + c3 − 3c (u + c) (v + c) ≥ M (u + c) (v + c)2 + (v + c) c2 + c(u + c)2 − 3c (u + c) (v + c) ⇔ (3 − M ) u2 − uv + v + u3 + v − M uv ≥ Bất đẳng thức cuối ln vì, − M > 0, u2 − uv + v ≥ uv ≥ u3 + v = u3 + v3 v3 + ≥ √ uv = M uv 2 Từ ta hồn tất phần chứng minh Vậy giá trị lớn cần tìm là: M = √ Câu 2.4 Vì tính bất đẳng thức nên ta chuẩn hóa xyz = Từ ta biến đổi: a b c x = , y = , z = Khi bất đẳng thức trở thành: b c a a2 b2 c2 a b c + + + 3k ≥ (k + 1) + + bc ca ab b c a ⇔ a3 + b3 + c3 + 3k · abc ≥ (k + 1) ab2 + bc2 + ca2 ⇔ a3 + b3 + c3 − 3abc ≥ (k + 1) ab2 + bc2 + ca2 − 3abc 160 BÀI TỐN TÌM HẰNG SỐ TỐT NHẤT Đây bất đẳng thức 3 ⇔ k = −1 + √ Vậy k + = √ 3 4 Câu 2.5 Ta chọn b = c giả thiết trở thành: a + 2b = b2 + 2ab ⇒ a = Vì a > nên 2b − b2 2b − 2b − b2 > ⇔ < b ≤ 2b − Bất đẳng thức trở thành: (a + 2b) + 2b a + b  ≥ k (a + 2b + 1)  1  2b − b2 ≥k + 2b  +   2b − b 2b − 2b +b 2b − 3b − b 2k ⇔ ≥ 3b + 5b + b − ⇒ 2b − b2 + 2b + 2b − 1 3b2 − b ;2 Xét hàm số f (b) = 3b + 5b2 + b − Ta có − (b2 − 1) (3b − 1)2 f (b) = ⇔ = ⇔ b = (3b3 + 5b2 + b − 1)2 Bằng cách lập bảng biến thiên ta tìm minf (b) = b = Từ tìm k ≤  ;2 Ta chứng minh k = giá trị lớn cần tìm Thực ta cần chứng minh: 1 + + ≥a+b+c+1 a+b b+c c+a 1 ⇔ (ab + bc + ca) + + ≥a+b+c+1 a+b b+c c+a ab bc ca ⇔ + + ≥ a+b b+c c+a (a + b + c) Nhận thấy dấu “=” đạt biến hai biến nên đánh giá bất đẳng thức thông thường có dấu “=” tâm khơng dẫn đến kết Để đảm bảo dấu “=” ta loại bỏ dấu “=” biên cách sử dụng abc ≥ sau: Theo Cauchy - Schwarz: ab bc ca (ab + bc + ca)2 + + ≥ a+b b+c c+a ab (a + b) + bc (b + c) + ca (c + a) (ab + bc + ca)2 = (a + b + c) (ab + bc + ca) − 3abc (ab + bc + ca)2 ≥ = (a + b + c) (ab + bc + ca) Chứng minh hoàn thành Ta kết luận số k lớn cần tìm k = 161 BÀI TỐN TÌM HẰNG SỐ TỐT NHẤT Câu 2.6 Cho b = c, ta có bất đẳng thức trở thành: + a b (a + 2b) −9≥k 1− 2ab + b2 a2 + 2b2 (a − b)2 2(a − b)2 ≥k· ab a + 2b2 k a2 + 2b2 ≥ ⇔ ab ⇔ Theo bất đẳng thức AM – GM, ta có: √ a2 + 2b2 ≥ 2 ab √ Suy k ≤ Ta chứng minh giá trị k lớn cần tìm Sử dụng bất đẳng thức Schur – SOS sau: √ 1 ab + bc + ca + + −9≥4 1− a b c a + b2 + c2 √ a2 + b2 + c2 − ab − bc − ca (a + b + c) (ab + bc + ca) − 9abc ≥4 2· ⇔ abc a2 + b2 + c2 √ (a − b)2 + (a − c) (b − c) 2c(a − b)2 + (a + b) (a − c) (b − c) ⇔ ≥4 2· abc a2 + b2 + c2 ⇔ M · (a − b)2 + N · (a − c) (b − c) ≥ √ √ 2c a+b Trong đó: M = − ,N= − abc a2 + b2 + c2 abc a + b2 + c2 Giả sử c = max {a,b,c} việc hồn tất N ≥ 0, hay √ (a + b) a2 + b2 + c2 ≥ 2abc (a + b + c) Do tính nên ta chuẩn hóa a + b + c = Ta chứng minh kết mạnh sau: √ √ a2 + b2 + c2 ≥ c (a + b) ⇔ − 2c (a + b) ≥ c (a + b) + 2ab Và mạnh nữa: √ (a + b)2 2 √ t ⇔ − · t (1 − t) ≥ · t (1 − t) + , với t = a + b √ √ √ −2 + t 3−2 2 ⇔ t − 2+ t+1≥0⇔ −1 2 − · c (a + b) ≥ · c (a + b) + ≥ Điều Như chứng minh hoàn √ tất Vậy số k lớn thỏa đề k = Câu 2.7 Ta chọn a = b = t, a = b = t, c = − 2t Khi đó, ta có: k 2t4 + (3 − 2t)4 − ≥ 2t3 + (3 − 2t)3 + 3t2 (3 − 2t) − −12t3 + 45t2 − 54t + 21 ⇔k≥ 18t4 − 96t3 + 216t2 − 216t + 78 3(t − 1)2 (7 − 4t) = (t − 1)2 (18t2 − 60t + 78) − 4t = = f (t) 2 (3t − 10t + 13) 162 BÀI TOÁN TÌM HẰNG SỐ TỐT NHẤT Ta có: 12 (2t2 − 7t + 3) 3; = ⇔ t ∈ 2 4(3t − 10t + 13) Khoảng giá trị t (0; 3), từ ta thấy minf (t) = t = (0;3) Vậy trở lại toán, ta cho a = b = , c = ta k ≥ 27 , ta chứng minh giá trị nhỏ 2 k cách chứng minh: f (t) = a + b4 + c4 − ≥ a3 + b3 + c3 + 3abc − Đặt f (a,b,c) = V T − V P Ta chứng minh: f (a,b,c) ≥ f ⇔ a+b a+b , ,c 2 (a + b)4 (a + b)3 (a + b)2 a + b4 − − a3 + b3 − − 3c ab − 4 (a − b)4 + 6(a − b)2 (a + b)2 (a + b) (a − b)2 (a − b)2 ⇔ − + 3c · ≥0 28 4 7a2 + 7b2 + 10ab 3c ⇔ − (a + b) + ≥0 28 4 ⇔ a2 + b2 + 10ab − 21 (a + b) + 21c ≥ ⇔ a2 + b2 + 10ab − 21 (a + b) + 21 (3 − a − b) ≥ ⇔ (a + b)2 + a2 + b2 − 42 (a + b) + 63 ≥ Do (a2 + b2 ) ≥ (a + b)2 nên ta chứng minh: 6(a + b)2 − 42(a + b) + 63 ≥ Và điều ta giả sử c = max {a,b,c} a + b ≤ Vậy ta cần ra: f ⇔ a+b a+b , ,c 2 ≥ ⇔ f (t; t; − t) ≥ 0,t = a+b 2 2t4 + (3 − 2t)4 − ≥ 2t3 + (3 − 2t)3 + 3t2 (3 − 2t) − Bất đẳng thức theo lập luận chặn t phía Vậy k = giá trị nhỏ cần tìm Câu 2.8 Cho b = c giả thiết cho viết thành: a + 2b = b2 + 2ab ⇒ a = b2 − 2b ≥ ⇒ ≤ b ≤ − 2b Lưu ý tìm số thực k lớn nên xét k > Thay vào bất đẳng thức, ta được: b2 − 2b b2 (b2 − 2b) +k ≥k+3 − 2b − 2b ⇔ 2b (1 − 2b) + b2 − 2b + kb2 b2 − 2b ≥ k (1 − 2b) + (1 − 2b) ⇔ (b − 1)2 ≥ k b2 − (b − 1)2 ⇔ ≥ b2 − k 2b + 163 ≥0 BÀI TỐN TÌM HẰNG SỐ TỐT NHẤT Mà b2 − ≤ 22 − = suy k ≤ Ta chứng minh k = giá trị lớn cần tìm Với giả thiết a + b + c = ab + bc + ca, ta cần chứng tỏ ab + bc + ca + abc ≥ Ta phản chứng ab + bc + ca + abc = chứng minh a + b + c ≥ ab + bc + ca Đây kết quen thuộc kì thi VMO 1996 Câu 2.9 Trong bất đẳng thức ban đầu, ta cho b = c, ta được: a3 + 2b3 b2 + 2ab k +k· + ≥ 2b (a + b) (a + 2b) a3 + 2b3 1 b2 + 2ab ⇔ − ≥k· − b(a + b)2 (a + 2b)2 (a − b)2 (a − b)2 (4a + 5b) ≥ k · ⇔ 8b(a + b)2 3(a + 2b)2 8k (a + 2b)2 (4a + 5b) ⇔ ≤ b(a + b)2 Do tính nên ta chọn b = Khi đó, ta có: 8k (a + 2)2 (4a + 5) ≤ (a + 1)2 (a + 2)2 (4a + 5) Ta xét hàm f (a) = , a > Ta có: (a + 1)2 √ −1 + (a + 2) (2a2 + 2a − 1) =0⇔a= f (a) = (do a > 0) (a + 1)3 √ √ √ 9+6 −1 + Lập bảng biến thiên ta f (a) ≥ f = + Suy ra: k ≤ √ 9+6 Ta chứng minh giá trị k lớn cần tìm Ta sử dụng kĩ thuật Schur – SOS phần chứng minh Bất đẳng thức cần chứng minh viết dạng: (a3 + b3 + c3 − 3abc) − ((a + b) (b + c) (c + a) − 8abc) a2 + b2 + c2 − ab − bc − ca ≥k· (a + b) (b + c) (c + a) 3(a + b + c)2 Ta sử dụng khai triển: a3 + b3 + c3 − 3abc = (a + b + c) (a − b)2 + (a − c) (b − c) (a + b) (b + c) (c + a) − 8abc = 2c (a − b)2 + (a + b) (a − c) (b − c) a2 + b2 + c2 − ab − bc − ca = (a − b)2 + (a − c) (b − c) Từ ta nhóm điều cần chứng minh thành: M (a − b)2 + N (a − c) (b − c) ≥ 0, đó: 8a + 8b + 2c k − (a + b) (b + c) (c + a) 3(a + b + c)2 5a + 5b + 8c k N= − (a + b) (b + c) (c + a) 3(a + b + c)2 M= 164 BÀI TỐN TÌM HẰNG SỐ TỐT NHẤT Bây giờ, không giảm tổng quát ta giả sử c = {a,b,c} nên a + b ≥ 2c Từ 8a + 8b + 2c ≥ 5a + 5b + 8c Từ ta thấy M ≥ N Hơn có (a − c) (b − c) ≥ Hơn có (a − c) (b − c) ≥ Như bất đẳng thức chứng minh hoàn tất ta N ≥ Tức là: (5a + 5b + 8c) (a + b + c)2 ≥ 8k (a + b) (b + c) (c + a) Đổi biến (a + b,b + c,c + a) → (X,Y,Z) Khi cần chứng minh: (X + 4Y + 4Z) (X + Y + Z)2 ≥ 32k · XY Z Ta chứng minh kết mạnh là: √ (X + 4Y + 4Z) (X + Y + Z)2 ≥ 8k · X (Y + Z)2 = 27 + 18 X (Y + Z)2 Vì tính nên ta chuẩn hóa X + Y + Z = Từ ta cần chứng minh: √ (4 − 3X) ≥ 27 + 18 X (1 − X)2 ⇔ g (X) = √ − 3X 3, ≥ + X(1 − X)2 < X < √ Khảo sát hàm g khoảng (0; 1) ta g (X) ≥ + Như ta hoàn tất chứng minh √ 9+6 Vậy k lớn Câu 2.10 Cho b = c ta được: a +k 2b b +k b+a ≥ k+ Cho a → ta được: √ −1 − k ≤ 4√ 8k (k + 1)2 ≥ (2k + 1)3 ⇔ 4k + 2k ≥ ⇔   −1 + k≥  Ta chứng minh toàn giá trị k cần tìm Đặt x = 2b 2c 2a ,y= ,z= b+c c+a a+b xy + yz + zx + xyz = Ta cần chứng minh: (x + 2k) (y + 2k) (z + 2k) ≥ (2k + 1)3 ⇔ xyz + 2k (xy + yz + zx) + 4k (x + y + z) + 8k ≥ (2k + 1)3 Theo kết VMO 1996, ta được: x + y + z ≥ xy + yz + zx Từ đó: xyz + 2k (xy + yz + zx) + 4k (x + y + z) + 8k ≥ xyz + 2k (xy + yz + zx) + 4k (xy + yz + zx) + 8k = (xy + yz + zx + xyz) + 4k + 2k − (xy + yz + zx) + 8k = 8k + + 4k + 2k − (xy + yz + zx) 165 BÀI TỐN TÌM HẰNG SỐ TỐT NHẤT Hơn ta có: xy + yz + zx ≥ 3 (xyz) ⇒ xyz ≤ (xy + yz + zx)3 √ 3 Từ đó: = xy + yz + zx + xyz ≤ xy + yz + zx + với t = (xy + yz + zx)3 t3 √ = t2 + √ , 3 3 √ √ xy + yz + zx Suy ra: t ≥ 3⇒ xy + yz + zx ≥ Suy ra: xyz + 2k (xy + yz + zx) + 4k (x + y + z) + 8k ≥ 8k + + 4k + 2k − (xy + yz + zx) ≥ 8k + + 4k + 2k − = (2k + 1)3 Bài tốn hồn tất Câu 2.11 Trước hết ta tìm điều kiện cần cho k Thay a = b = x, c = với x > 0, x = 1, bất đẳng thức cho viết lại thành x k k ≥3+ 2x + + x x2 − ⇔ x2 + − ≥ k x 2x + x2 + x2 + + x (x − 1)2 (x + 2) k(x − 1)2 (2x + 1) ≥ x (2x3 + x2 + 1) k (x + 2) (2x3 + x2 + 1) ⇔ ≤ x (2x + 1) k ⇔ ≤ x2 + 2x + − , ∀x > 0, x = x 2x + ⇔ √ 3+1 Bây giờ, ta cho x = tính giá trị biểu thức f (x) = x2 + 2x + − x 2x + bên vế phải Để ý x nghiệm phương trình, dó ta có x2 = = − 4x Suy 2x + 4x + 1 , = 8x − 4, x 4x + + 2x + (8x − 4) − (4 − 4x) √ √ 95 45 45 − 50 45 + 95 = x− = · − = 8 f (x) = Với kết này, ta thu √ √ 45 − 50 k≤ = 45 × − 25 ≈ 45 × 0.866 − 25 ≈ 13.97 2 166 BÀI TỐN TÌM HẰNG SỐ TỐT NHẤT Mặt khác, k số nguyên nên từ có k ≤ 13 Tiếp theo, ta chứng minh k = 13 thỏa mãn yêu cầu toán, tức 1 13 25 + + + ≥ a b c a+b+c+1 Đặt f (a,b,c) = 1 13 + + + a b c a+b+c+1 Khơng tính tổng quát, giả sử a = max{a,b,c}, ta có √ √ f (a,b,c) − f a, bc, bc = 1 1 √ + −√ + 13 − b c a + b + c + a + bc + bc   √ √ 13  = b− c  − bc (a + b + c + 1) a + 2√bc + Do a = max{a; b; c} giả thiết abc = nên ta có bc ≤ 1, suy ≥ Mặt khác, sử dụng bất bc đẳng thức AM - GM, ta lại có 13 ≤ √ (a + b + c + 1) a + bc + 13 √ 3 abc + √ 3 abc + = 13 < 16 Từ ta đưa toán chứng minh f ,x,x x2 ≥ 25 √ với x = bc, < x ≤ Nếu x = bất đẳng thức trở thành đẳng thức Trong trường hợp < x < cách sử dụng lại biến đổi thực trình tìm điều kiện cho k, ta thấy bất đẳng thức tương đương với 13 (x + 2) (2x3 + x2 + 1) ≥ x (2x + 1) ⇔ (x + 2) 2x + x + ≥ 13x (2x + 1) ⇔ 2x4 + 5x3 + 2x2 + x + ≥ 26x2 + 13x ⇔ 8x4 + 20x3 − 18x2 − 9x + ≥ Ta có 8x4 + 20x3 − 18x2 − 9x + = 8x4 − 8x2 + + 20x3 − 20x2 + 5x + 10x2 − 14x + = 2x2 − + 5x (2x − 1)2 + 5x2 − 7x + > Do (2x2 − 1) ≥ 0,5 · x (2x − 1)2 ≥ 5x2 − 7x + > (tam thức bất hai có hệ số cao dương biệt thức ∆ = −11 < 0) Như vậy, bất đẳng thức cuối hiển nhiên Vậy k = 13 giá trị cần tìm 167 BÀI TỐN TÌM HẰNG SỐ TỐT NHẤT Câu 2.12 Cho a = b = t > 0, c = ta t2 2t6 + t3 + 2 + t ⇔ + 3k ≥ (k + 1) + 3k ≥ (k + 1) · t4 t t4 t 6 3 t + 2t + −t + 2t3 − t +2 3t − t − ≥ − ≥ ⇔ k 3− ⇔ k t t t4 t t4 2t2 + (t3 − 1) ⇔ k (t − 1) (t + 2) ≤ t3 2 (t + t + 1) ⇔ k (t + 2) ≤ t3 2 ⇔k≤ (t2 + t + 1) ⇔k≤ t4 + 2t3 1 + t t 1+ t 1+ Đặt z = Khi t k≤ (z + z + 1) 2z + Khi t → +∞ z → Như kmax = f (z) , [0;+∞) với f (z) = (z + z + 1) , với z ≥ Ta có 2z + (z + z + 1) (2z + 1)2 − (z + z + 1) (2z + 1)2 (z + z + 1) (2z + 1)2 − = ≥ 0, ∀z ≥ (2z + 1)2 f (z) = Suy hàm số f đồng biến [0; +∞), với z ≥ f (z) ≥ f (0) = Do k ≤ Từ kmax = 1 + + a b c ⇔ a2 + b2 + c2 + ≥ (ab + bc + ca) (do abc = 1) ⇔ a2 + b2 + c2 + 2abc + ≥ (ab + bc + ca) a2 + b2 + c2 + ≥ Đây kết quen thuộc chứng minh ngun lí Diricle (ta dùng dồn biến để chứng minh) Dấu đẳng thức xảy (a,b,c) = (1; 1; 1); a = b → +∞, c → hoán vị Câu 2.13 Thơng thường dạng tốn này, đẳng thức đạt ngồi giá trị tâm (1; 1; 1) đạt hai biến Như vạy để chặn k, ta chọn a = b = t, c = 3−2t Ta có: + − ≥ k 2t2 + (3 − 2t)2 − t − 2t − 4t + t − (3t − 2t2 ) ⇔ ≥ k 6t2 − 12t + 3t − 2t2 6t2 − 12t + ⇔k≤ ⇔k≤ 2 (3t − 2t ) (6t − 12t + 6) t (3 − 2t) 168 BÀI TỐN TÌM HẰNG SỐ TỐT NHẤT Như giá trị c = − 2t lớn cần tìm giá trị lớn hàm số f (t) = ,t∈ t (3 − 2t) 0; Theo bất đẳng thức AM - GM, ta có t (3 − 2t) = Từ kết luận kmax = 1 (2t + − 2t)2 · 2t (3 − 2t) ≤ · = 2 8 chứng tỏ 1 + + −3≥ a + b2 + c2 − a b c Đặt f (a,b,c) = (1) 1 + + − − (a2 + b2 + c2 − 3) Xét hiệu a b c a+b a+b , ,c 2 1 = + + −3− a + b2 + c2 − − a b c (a + b)2 − − +3+ + c2 − a+b c f (a,b,c) − f = 1 + − a b a+b − a2 + b2 − (a + b)2 (a + b)2 − 4ab a2 + b2 − 2ab = − · ab (a + b) = (a − b)2 − ab (a + b) Ta giả sử c = max {a,b,c} Khi a + b ≤ 2, suy ab (a + b) ≤ (a + b)3 ≤ < 4 f (a,b,c) ≥ f a+b a+b , ,c 2 Như Ta cần chứng minh f a+b a+b , ,c 2 ≥ ⇔ f (t,t,3 − 2t) ≥ 0, với t = a+b Tức chứng minh + −3≥ 2t2 + (3 − 2t)2 − t − 2t 24t − 16t2 − ⇔ ≤ ⇔ ≤0 t (3 − 2t) t (3 − 2t) 16t2 − 24t + (4t − 3)2 ⇔ ≥0⇔ ≥ (đúng) t (3 − 2t) t (3 − 2t) Vậy (∗) chứng minh Từ kết luận kmax = 169 BÀI TỐN TÌM HẰNG SỐ TỐT NHẤT Câu 2.14 Kí hiệu (1) bất đẳng thức nêu đề Giả sử k số thực cho bất đẳng thức (1) với ba số thực a, b, c, mà abc ≥ Trong (1) thay a = 0, b = c = 2, ta + 8k ≥ Suy k ≥ Ta chứng minh k = giá trị nhỏ cần tìm, tức chứng minh với ba số thực không âm a, b, c, ta ln có abc + (a − b)2 + (b − c)2 + (c − a)2 + ≥ a + b + c hay a2 + b2 + c2 + 2abc + ≥ ab + bc + ca + (a + b + c) (2) Xét số (b − 1) (c − 1), (c − 1) (a − 1), (a − 1) (b − 1), ta có a (b − 1) (c − 1) · b (c − 1) (a − 1) · c (a − 1) (b − 1) = abc (a − 1)2 (b − 1)2 (c − 1) ≥ Suy có số số nêu khơng âm Khơng tính tổng qt, giả sử a (b − 1) (c − 1) ≥ Khi đó, ta có abc ≥ a (b + c − 1) Do vậy, bất đẳng thức (2) chứng minh, ta chứng minh a2 + b2 + c2 + 2a (b + c − 1) + ≥ ab + bc + ca + (a + b + c) , hay a2 − (4 − b − c) a + b2 + c2 − bc − (b + c) + ≥ 0, (3) với a, b, c, mà abc ≥ Ta xem vế trái (3) tam thức bậc hai theo ẩn a, ta có: ∆ = (b + c − 4)2 − 4b2 − 4c2 + 4bc + (b + c) − 16 = −3b2 − 3c2 + 6bc = −3 (b − c)2 ≤ 0, với b, c nên (3) với a, b, c, mà abc ≥ Vậy k = giá trị cần tìm, theo yêu cầu đề Câu 2.15 Rõ ràng, giá trị x = y = z = thỏa mãn ràng buộc nêu đề Vì thế, bất đẳng thức đề bài, cho x = y = z = ta k + ≥ Suy k ≥ Tiếp theo, ta chứng minh với k = 1, bất đẳng thức đề bất đẳng thức đúng; tức ta chứng minh x+y+z 2 (x + 1) (y + 1) (z + 1) ≤ + 1, (1) với số thực dương x, y, z mà {xy,yz,zx} ≥ Thật vậy, trước hết, ta chứng minh nhận xét sau: Nhận xét: Với a, b hai số thực dương thỏa mãn ab ≥ 1, ta ln có: a+b a2 + b + ≤ +1 Chứng minh: Vì ab ≥ nên a +1 2 a+b 2 − ≥ ab − ≥ Do b + = (ab − 1) + (a + b) ≤ a+b 170 2 −1 + (a + b) = a+b 2 +1 BÀI TỐN TÌM HẰNG SỐ TỐT NHẤT Nhận xét chứng minh Không tổng quát, giả sử x ≤ y ≤ z Khi đó, từ ràng buộc đề bài, suy x ≥ x+y+z Đặt t = Ta có xt = x2 + xy + xz 1+1+1 x (x + y + z) = ≥ = 3 Do đó, áp dụng nhận xét cho cặp (x,t) cặp (y,z), ta 2 t +1 ≤ x +1 x+t 2 y+z 2 +1 2 z +1 ≤ y +1 , (2) (3) +1 Nhân (2) (3), vế theo vế, ta x +1 Nhận thấy 2 z +1 y +1 t +1 ≤ x+t 2 +1 y+z 2 +1 (4) x+t y+z √ √ · ≥ xt · yz ≥ Do đó, theo nhận xét, ta có 2 x+t 2 +1 2 y+z +1 ≤ x+y+z+t 4 +1 (5) Từ (4) (5), suy x2 + y2 + z2 + t2 + ≤ t2 + 1) (t2 x+y+z Do (x2 + 1) (y + 1) (z + + 1) ≤ + Suy (1) chứng minh thế, giá trị k nhỏ cần tìm theo yêu cầu k = Câu 2.16 Kí hiệu (1) bất đẳng thức nêu đề Giả sử k số thực cho bất đẳng thức (1) với ba số thực a, b, c độ dài ba cạnh tam giác Trong (1) thay b = c > ta a 2b 2ab + b2 + +k· ≤ + k 2b a + b a + 2b2 k ≤ + k Suy k ≥ 2 Ta chứng minh k = giá trị nhỏ cần tìm, tức chứng minh với ba số thực a, b, c độ dài ba cạnh tam giác, ta ln có Cố định b cho a tiến tới 0+ ta + a b c ab + bc + ca + + + ≤ b + c c + a a + b a + b2 + c2 171 (2) BÀI TỐN TÌM HẰNG SỐ TỐT NHẤT Thật vậy, ta có: (2) ⇔ 1− a b+c + 1− b c+a + 1− c a+b ≥ ab + bc + ca + a2 + b2 + c2 b+c−a c+a−b a+b−c (a + b + c)2 ⇔ + + ≥ b+c c+a a+b (a2 + b2 + c2 ) (b + c − a)2 (c + a − b)2 (a + b − c)2 (a + b + c)2 ⇔ + + ≥ (b + c) (b + c − a) (c + a) (c + a − b) (a + b) (a + b − c) (a2 + b2 + c2 ) (3) Do a, b, c độ dài ba cạnh tam giác nên b + c − a > 0, c + a − b > 0, a + b − c > Do tất phân thức nằm vế trái (3) có mẫu thức dương Vì thế, kí hiệu V T biểu thức nằm vế trái (3), theo bất đẳng thức Cauchy – Schwarz dạng Engel, ta có: VT ≥ (a + b + c)2 (a2 + b2 + c2 ) Vì (b + c − a) + (c + a − b) + (a + b − c) = a + b + c (b + c − a) (b + c) + (c + a − b) (c + a) + (a + b − c) (a + b) = a2 + b2 + c2 Nên (3) chứng minh (2) chứng minh Vậy k = giá trị cần tìm, theo yêu cầu đề thỏa mãn ràng buộc nêu đề Vì thế, 9k + ≤ , mà k > nên suy bất đẳng thức đề bài, cho x = y = z = ta 3k + 2 k ≥ Tiếp theo, ta chứng minh với k = , bất đẳng thức đề bất đẳng thức đúng; tức ta chứng minh 1 + + ≤ 3, (1) 1 x+y+ y+z+ z+x+ 3 √ √ √ với số thực dương x, y, z mà xy + yz + zx = Thật vậy, đặt √ √ √ T = x + y + z = x + y + z + Câu 2.17 Rõ ràng, giá trị x = y = z = Áp dụng bất đẳng thức Cauchy-Schwarz cho hai số dương, ta có x+y+ 1 + +z 3 ≥ √ 1 √ √ x· √ + y· √ + √ · z 3 hay x+y+ ≤ 3z + T Tương tự ta có: y+z+ ≤ 3x + ; T z+x+ 172 ≤ 3y + T = T BÀI TỐN TÌM HẰNG SỐ TỐT NHẤT Cộng bất đẳng thức vừa nêu trên, vế với vế, với lưu ý T = x + y + z + 2, ta x+y+ + y+z+ + z+x+ 3z + + 3x + + 3y + = T ≤ (1) chứng minh Vậy k = giá trị cần tìm, theo yêu cầu đề Câu 2.18 Kí hiệu (1) bất đẳng thức cần chứng minh Trong (1) cho a = b = c = 1, suy k ≤ 729 Ta chứng minh (1) với k = 729 Từ giả thiết ta có 3 |abc| ≥ |ab| + |bc| + |ca| ≥ (abc)2 ⇒ abc ≥ Do ≥ a2 + |bc| c2 + a2 + b2 b2 + a2 + c2 a2 + b2 + c2 b2 + |ac| c2 + |ab| ≥ 93 (abc)2 ≥ 729 Sử dụng bất đẳng thức Cauchy cho chín số a2 số |bc| Vậy max k = 729 Câu 2.19 Gọi (1) bất đẳng thức cần tìm Trong (1) cho a1 = 1; a2 = · · · = an = − ta n−1 n c ≤ n Ta chứng minh BĐT (1) với c = Tức cần chứng minh n |ai − aj | ≥ 1≤i

Ngày đăng: 08/12/2022, 23:19

Từ khóa liên quan

Tài liệu cùng người dùng

  • Đang cập nhật ...

Tài liệu liên quan